You are on page 1of 90

Most Probable

Prelims Questions
Compilation of ClearIAS Daily MCQs

Set - 5

Sourced from the repository of questions used


for the ClearIAS Email Program
1. With reference to the United Nations General Assembly (UNGA),
consider the following statements

1. The decisions are taken through a vote in UNGA.


2. The assembly has binding votes or Veto powers.

Which of the statements given above is/are correct?

(a) 1 only
(b) 2 only
(c) Both 1 and 2
(d) Neither 1 nor 2

Ans: (a) 1 only.

Learning Zone: Popularly known as the parliament of the world, where all the 193
UN member states are represented, the UNGA is the deliberative, policymaking and
representative organ of the UN.

It Takes a decision on important matters such as peace and security and discusses
various global issues and budgetary matters.

Decisions are taken through a vote. Admission of new members and budgetary
matters require a two-thirds majority, while the decision on other issues are taken
by a simple majority.

Why is this question important? Each sovereign state gets one vote and the
votes are not binding on the membership, except in budgetary matters.

The Assembly has no binding votes or veto powers like the UN Security Council.

2. With reference to UN Global Climate Action Awards, consider the


following statements

1. It is spearheaded by Momentum for a Change initiative,


1
Page
2. Infosys has won the United Nations Global Climate Action Award from India
this year.

Which of the statements given above is/are correct?

(a) 1 only
(b) 2 only
(c) Both 1 and 2
(d) Neither 1 nor 2

Ans: (c) Both 1 and 2

Learning Zone: The Awards are spearheaded by the Momentum for Change
initiative at UN Climate Change.

The projects are recognized as innovative solutions that not only address climate
change but also help drive forward progress on many other sustainable development
goals, for example, innovation, gender equality and economic opportunity.

Why is this question important? From India, IT major Infosys has won the
United Nations Global Climate Action Award (UNGCAA) in the ‘Climate Neutral Now’
category for “Infosys’ Journey to Carbon Neutrality”.

3. Consider the following statements with regard to the Supreme Court of


India

1. The government can relocate the Supreme court from Delhi to any other
parts of the country.
2. The President can constitute the benches of the Supreme court.

Which of the statements given above is/are correct?

(a) 1 only
(b) 2 only
(c) Both 1 and 2
(d) Neither 1 nor 2
2
Page
Ans: (d) Neither 1 nor 2

Learning Zone: Article 130 says that “the Supreme Court shall sit in Delhi or in
such other place or places, as the Chief Justice of India may, with the approval of
the President, from time to time, appoint.”

Why is this question important? Supreme Court Rules give the Chief Justice of
India the power to constitute Benches — he can, for instance, have a Constitution
Bench of seven judges in New Delhi, and set up smaller Benches in, say, four or six
places across the country.

4. Sixth Assessment Report (AR6), recently seen in the news is published


by which among the following?

(a) IUCN
(b) IPCC
(c) World Economic Forum
(d) IMF

Ans: (b) IPCC

Learning Zone: IPCC Working Group III is responsible for assessing the mitigation
of climate change – responses and solutions to the threat of dangerous climate
change by reducing emissions and enhancing sinks of the greenhouse gases that are
responsible for global warming.

Comprehensive scientific assessment reports are published every 6 to 7 years; the


latest, the Fifth Assessment Report, was completed in 2014 and provided the main
scientific input to the Paris Agreement.

Why is this question important? IPCC Working Group III is meeting in India to
further preparations for the Sixth Assessment Report.

More than 200 experts from 65 countries will come together to start preparing a first
draft of the report, which is due to be finalized in July 2021. The meeting is hosted
by the Ministry of Environment, Forest & Climate Change, Government of India.
3
Page
5. ‘Industry 4.0’, a Pilot Project for implementation at Modern Coach
Factory, Raebareli, has been initiated by which among the following?

1. World Economic Forum


2. Department of Science & Technology
3. IIT Kanpur
4. Ministry of Railways

Select the correct answer using the codes given below:

(a) 1 and 2 only


(b) 2, 3 and 4only
(c) 2 and 4 only
(d) 1 and 4 only

Ans: (b) 2, 3 and 4 only

Learning Zone: The Ministry of Railways and Department of Science & Technology
have joined hands in partnership with IIT Kanpur for taking up a unique project on
‘Industry 4.0’ by launching a Pilot Project for implementation at Modern Coach
Factory, Raebareli.

Why is this question important? Full transition to the digital factory using
‘Industry 4.0’ across entire value chain from design to production will help enhance
productivity hugely by providing insight into production process to make the
decisions in real-time basis, minimizing human errors by effective monitoring to
ensure that resources are put to the best utilization measured by, what is called the
Overall Equipment Effectiveness (OEE).

6. With reference to INS Nilgiri, recently seen in the news consider the
following statements

1. It is the first ship of Project 17A.


2. It is the Indian Navy’s first new stealth frigate.

Which of the statements given above is/are correct?


4
Page
(a) 1 only
(b) 2 only
(c) Both 1 and 2
(d) Neither 1 nor 2

Ans: (c) Both 1 and 2

Learning Zone: Project 17A frigates is a design derivative of the Shivalik class
stealth frigates with much more advanced stealth features and indigenous weapons
and sensors.

These frigates are being built using integrated construction methodology.

The Nilgiri-class frigate or Project 17A is a follow-on of the Project 17 Shivalik-class


frigate for the Indian Navy. A total of seven ships will be built at Mazagon Dock and
GRSE.

The ships have been named after the first major warships to be built in India, which
in turn were named after hill ranges in India.

Why is this question important? The construction of the first ship started in
2017 and the first ship is expected to be delivered by 2022.

7. With reference to the National Nutrition Survey, recently published by


the Ministry of Health and Family Welfare, consider the following
statements

1. It is conducted with the help of UNICEF.


2. It is the first-ever comprehensive National Nutrition Survey.
3. According to survey Tamil Nadu and Goa had the highest number of
adolescents who were obese or overweight.

Which of the statements given above is/are correct?

(a) 1 and 2 only


(b) 2 and 3 only
5

(c) 1 and 3 only


Page
(d) 1, 2 and 3

Ans: (d) 1, 2 and 3

Learning Zone: With the help of UNICEF, the Ministry of Health and Family Welfare
recently conducted the first-ever comprehensive National Nutrition Survey.

The survey recorded malnutrition that included micronutrient deficiencies and details
of non-communicable diseases such as diabetes, hypertension, cholesterol and
kidney function in children and adolescents.

Around 10% of children in the age group of 5 to 9 years and adolescents in the age
group 10 to 19 years are pre-diabetic. 5% of them were overweight and 5%
suffered from blood pressure.

Tamil Nadu and Goa had the highest number of adolescents who were obese or
overweight.

Why is this question important?


The survey for the first time proved the coexistence of obesity and undernutrition.
One in five children in the age group 5 to 9 years were stunted.

8. School Education Quality Index (SEQI), recently seen in the news is


released by which among the following?

1. NITI Aayog
2. UNICEF
3. Ministry of Human Resource Development
4. CBSE

Select the correct answer using the codes given below:

(a) 1 only
(b) 2 and 3 only
(c) 1, 3 and 4 only
6

(d) 1, 2, 3 and 4
Page
Ans: (a) 1 only

Learning Zone: Developed by NITI Aayog to evaluate the performance of States


and Union Territories (UTs) in the school education sector.

Why is this question important? It aims to bring an ‘outcomes’ focus on the


education policy by providing States and UTs with a platform to identify their
strengths and weaknesses and undertake requisite course corrections or policy
interventions.

9. With reference to Enterprise Development Centres (EDCs), consider the


following statements

1. It is an initiative of the Ministry of Micro, Small and Medium Enterprises.


2. It is to be launched in every district in the country.
3. It aims to offer enterprise development courses, vocational guidance and skill
development for budding entrepreneurs.

Which of the statements given above is/are correct?

(a) 1 and 2 only


(b) 2 and 3 only
(c) 1 and 3 only
(d) 1, 2 and 3

Ans: (d) 1, 2 and 3

Learning Zone: Union micro, small and medium enterprises sector (MSME)
ministry is planning to launch enterprise development centres (EDCs) in every
district.

Aimed at developing a cadre of indigenous entrepreneurs in the MSMEs, the EDCs


will be similar to incubators for start-ups.
7
Page
They shall be run by special purpose vehicles in partnership with the private sector,
business management organisations, local industry associations.

Why is this question important? It aims to Offer “enterprise development


courses, vocational guidance and skill development for budding entrepreneurs”.
Have “enterprise clinics” for struggling firms, which the government hopes will
reduce the number of small businesses falling into a debt trap financed by bank
loans.

10. With reference to recently held Drone Innovators Network Summit


2019, consider the following statements

1. It is organized by the World Economic Forum.


2. It was recently held in New Delhi this year.

Which of the statements given above is/are correct?

(a) 1 only
(b) 2 only
(c) Both 1 and 2
(d) Neither 1 nor 2

Ans: (c) Both 1 and 2

Learning Zone: Drone Innovators Network (DIN) is started by the World Economic
Forum to hasten and contribute to healthy drone policies in need of legislation.

It is organised by the World Economic Forum under the aegis of the Ministry of Civil
Aviation.

Established to help overcome common challenges regulators are facing in enabling


drones and unmanned aviation, such as how to enable beyond visual line of sight
(BVLOS) flights at scale, autonomous operations and flights over densely populated
areas.

Why is this question important? Drone Innovators Network Summit-2019 is


8

being held in New Delhi.


Page
11. With reference to Total Polar Compounds(TPC) often seen in the news,
consider the following statements

1. It is formed when the oil is frozen to a certain limit.


2. It is toxic in nature.
3. FSSAI has fixed a limit for Total Polar Compounds at 25%.
9

Which of the statements given above is/are correct?


Page
(a) 1 and 2 only
(b) 2 and 3 only
(c) 1 and 3 only
(d) 1, 2 and 3

Ans: (b) 2 and 3 only

Learning Zone: During frying, the oil undergoes degradation due to their exposure
to elevated temperatures which result in changes in the Physico-chemical, nutritional
and sensory properties of the oil. Polar compounds are formed during frying.

The toxicity of these compounds is associated with several diseases such as


hypertension, atherosclerosis, Alzheimer’s disease liver diseases etc. Therefore, it is
essential to monitor the quality of vegetable oils during frying.

In order to safeguard consumer health, FSSAI has fixed a limit for Total Polar
Compounds at 25% beyond which the vegetable oil shall not be used. Therefore,
Vegetable Oils should not be repeatedly used for frying.

Why is this question important? To mark the 150th birth anniversary of


Mahatma Gandhi on 2nd October as Rashtriya Swachhta Diwas, publicity vans were
flagged off in Delhi, to generate awareness among the people about the OMCs’
initiative of converting Used Cooking Oil to Biodiesel.

12. With regard to ‘The Gandhian Challenge’, recently launched by AIM,


NITI Aayog’s Atal Tinkering Labs (ATL) and UNICEF India, consider the
following statements

1. The challenge provides a platform for everyone across India to ideate


innovative solutions for a sustainable India of their dreams, using Gandhi’s
principles.
2. Ideas and solutions to the Gandhian Challenge may be expressed through
broad categories: Art & Innovation and Science, Technology & Innovation.
10

Which of the statements given above is/are correct?


Page
(a) 1 only
(b) 2 only
(c) Both 1 and 2
(d) Neither 1 nor 2

Ans: (b) 2 only

Learning Zone: This innovation challenge provides a platform for every child across
India to ideate innovative solutions for a sustainable India of their dreams, using
Gandhi’s principles.

The winners of The Gandhian Challenge will be awarded in New Delhi by NITI
Aayog’s Atal Innovation Mission and UNICEF on the occasion of Children’s Day in
November.

The contest – open for every child in India from 2 October to 20 October – also
celebrates 70 years of partnership between Government of India and UNICEF India
to enable Every Right for Every Child.

Why is this question important? On the 150th birth Anniversary of Mahatma


Gandhi, AIM, NITI Aayog’s Atal Tinkering Labs (ATL) and UNICEF India, including
Generation Unlimited, have launched ‘The Gandhian Challenge’.

13. D28 iceberg, recently seen in the news belongs to which among the
following places?

(a) Antarctica
(b) Arctic
(c) Pacific ocean
(d) South China Sea

Ans: (a) Antarctica

Learning Zone: It is about 210 metres thick and contains 315 billion tonnes of ice.
11

The east of Antarctica — where D28 broke off — is different from the west of the
continent and Greenland, which are rapidly warming due to climate change.
Page
Why is this question important? The iceberg, dubbed D28, broke away from the
Amery ice shelf, according to observations from European and American satellites.

14. Village Secretariat programme, recently seen in the news has been
initiated in which among the following states
12

(a) Kerala
(b) Andhra Pradesh
Page
(c) Karnataka
(d) Uttarakhand

Ans: (b) Andhra Pradesh

Learning Zone: Under the new system, the AP government, one Village Secretariat
has been set up for every population of 2,000, with each one comprising close to a
dozen village officials from various departments like police, revenue, etc.

The idea behind it is to ensure that its services reach people on the ground, and also
to strengthen the existing Panchayat Raj system.

The cost of hiring about 1.26 lakh new employees are going to be roughly about
₹2,200 crores a year for the AP government.

Why is this question important? The Andhra Pradesh government launched its
Village Secretariat programme, under which 1.26 lakh new government employees
will begin working.

15. Extended Producer Responsibility (EPR) scheme, often seen in the


news is related to which among the following?

(a) IPR Protection


(b) Plastic Waste Management Rules
(c) Bharat VI regulations
(d) Film Censoring

Ans: (b) Plastic Waste Management Rules

Learning Zone: Extended Producer Responsibility (EPR) scheme was introduced in


the Plastic Waste Management (PWM) Rules, 2011, and was largely redefined in
PWM 2016, wherein producers, importers and brand owners were asked to take
primary responsibility for collection of used multi-layered plastic sachets or pouches
or packaging.
13
Page
Why is this question important? The government has launched massive
campaigns against single-use, or disposable, plastic products, as part of the year-
long celebrations to mark the 150th birth anniversary of Mahatma Gandhi.

16. With reference to the National Health Systems Resource Centre


(NHSRC), consider the following statements

1. It is set up under the National Rural Health Mission (NRHM).


2. It is a WHO collaborating centre.

Which of the statements given above is/are correct?

(a) 1 only
(b) 2 only
(c) Both 1 and 2
(d) Neither 1 nor 2

Ans: (c) Both 1 and 2

Learning Zone: It is Set up under the National Rural Health Mission (NRHM) to
serve as an apex body for technical assistance.

Its mandate is to assist in policy and strategy development in the provision and
mobilization of technical assistance to the states and in capacity building for the
Ministry of Health and Family Welfare (MoHFW) at the centre and in the states.

Such global collaborations in the area of health technology will ensure that scientific
and technological advances, research and development as well as innovative
technologies play a substantial supportive role in healthcare and enable us to reach
the public health goals and achieve universal health coverage.

Why is this question important? National Health Systems Resource Centre


(NHSRC) designated as a WHO Collaborating Centre for Priority Medical Devices and
Health Technology Policy
14
Page
17. PRAKASH portal, recently launched by Government is related to which
among the following?

(a) Free electricity in all schools in the rural area


(b) Coordination in coal supplies to power plants
(c) Food delivery portal in Railways
(d) Solar panel distribution by the government

Ans: (b) Coordination in coal supplies to power plants

Learning Zone: PRAKASH (Power Rail Koyla Availability through Supply Harmony)
portal has been launched for transparency and better coordination in coal supplies to
power plants.

It aims at bringing better coordination for coal supplies among all stakeholders viz –
Ministry of Power, Ministry of Coal, Coal India, Railways and power utilities.

Why is this question important? Portal is developed by NTPC and sources data
from different stakeholders such as Central Electricity Authority (CEA), Centre for
Railway Information System (CRIS) and coal companies. All reports are available in
PDF/Excel format.

18. Youth Co: Lab, recently seen in the news, is an initiative launched by
which among the following?

1. Atal Innovation Mission (AIM)


2. NITI Aayog
3. UNDP India

Select the correct answer using the codes given below:

(a) 1 and 2 only


(b) 2 and 3 only
(c) 1 and 3 only
(d) 1, 2 and 3
15

Ans: (d) 1, 2 and 3


Page
Learning Zone: Youth Co: Lab will convene social innovation challenges at the
national and sub-national level, which will invite young people in the age group of
18-29 years and start-ups to showcase their proposed ideas and solutions to tackle
some of the region’s biggest social challenges.

Why is this question important? Atal Innovation Mission (AIM), NITI Aayog,
UNDP India Jointly Launch Youth Co: Lab to accelerate youth-led social
entrepreneurship and innovation in India.

19. With regard to the National Disaster Response Fund (NDRF), consider
the following statements

1. It is erstwhile known as National Calamity Contingency Fund (NCCF).


2. NDRF amount can be spent only towards meeting the expenses for
emergency response, relief and rehabilitation.
3. It is constituted to supplement the funds of the State Disaster Response
Funds (SDRF).

Which of the statements given above is/are correct?

(a) 1 and 2 only


(b) 2 and 3 only
(c) 1 and 3 only
(d) 1, 2 and 3

Ans: (d) 1, 2 and 3

Learning Zone: It is a fund managed by the Central Government for meeting the
expenses for emergency response, relief and rehabilitation due to any threatening
disaster situation or disaster.

Constituted to supplement the funds of the State Disaster Response Funds (SDRF) of
the states to facilitate immediate relief in case of calamities of a severe nature.
16

National Calamity Contingency Fund (NCCF) was renamed as National Disaster


Response Fund (NDRF) with the enactment of the Disaster Management Act in 2005.
Page
Why is this question important? NDRF amount can be spent only towards
meeting the expenses for emergency response, relief and rehabilitation.
For projects exclusively for the purpose of mitigation, i.e, measures aimed at
reducing the risk, impact or effect of a disaster or threatening disaster situation a
separate fund called National Disaster Mitigation Fund has to be constituted.

20. With reference to the Indian Council of World Affairs (ICWA), consider
the following statements

1. It is an institution of national importance.


2. The President of India is the ex-officio President of ICWA.
3. The founder-president of the Council was Sir Tej Bahadur Sapru.

Which of the statements given above is/are correct?

(a) 1 and 2 only


(b) 2 and 3 only
(c) 1 and 3 only
(d) 1, 2 and 3

Ans: (c) 1 and 3 only

Learning Zone: Established in 1943 by a group of Indian intellectuals as a think


tank.

By an Act of Parliament in 2001, the ICWA was declared an institution of national


importance.

The Vice President of India is the ex-officio President of ICWA.

The founder-president of the Council was Sir Tej Bahadur Sapru.

Why is this question important? It is devoted exclusively to the study of


international relations and foreign affairs.
17
Page
21. With reference to Information Fusion Centre (IFC) for the Indian
Ocean Region (IOR), consider the following statements

1. The IFC has been established at the Navy’s Information Management and
Analysis Centre (IMAC) in Gurugram.
2. Through this centre information on military shipping will be exchanged with
countries in the region to improve maritime domain awareness in the Indian
Ocean.

Which among the following is/are correct?

(a) 1 only
(b) 2 only
(c) Both 1 and 2
(d) Neither 1 nor 2

Ans: (a) 1 only

Learning Zone: The IFC-IOR was established with the vision of strengthening
maritime security in the region and beyond, by building a common coherent
maritime situation picture and acting as a maritime information hub for the region.

The IFC has been established at the Navy’s Information Management and Analysis
Centre (IMAC) in Gurugram, which is the single point centre linking all the coastal
radar chains to generate a seamless real-time picture of the nearly 7,500-km
coastline.

Why is this question important? Through this Centre, information on “white


shipping”, or commercial shipping, will be exchanged with countries in the region to
improve maritime domain awareness in the Indian Ocean.

22. ‘Geotail’, recently seen in the news refers to which among the
following?

(a) A region in space that allows the best observations.


18

(b) It is a satellite by NASA to study the Sun.


(c) An asteroid
Page
(d) Telescope by Israel to view the moon surface.

Ans: (a) A region in space that allows the best observations.

Learning Zone: It is a region in space that allows the best observations. The
region
exists as a result of the interactions between the Sun and Earth.

Why is this question important? An instrument on Chandrayaan-2, CLASS,


designed to detect signatures of elements in the Moon’s soil, had detected charged
particles during the mission. This happened in September, during the orbiter’s
passage through the “geotail”.

23. With reference to Multidisciplinary drifting Observatory for the Study


of Arctic Climate (MOSAiC), consider the following statements

1. It is Spearheaded by the Arctic Council.


2. It is the largest ever Arctic expedition in history.

Which of the statements given above is/are correct?

(a) 1 only
(b) 2 only
(c) Both 1 and 2
(d) Neither 1 nor 2

Ans: (b) 2 only

Learning Zone: It is spearheaded by the Alfred Wegener Institute in Germany.


It is the largest ever Arctic expedition in history.

It will be the first to conduct a study of this scale at the North Pole for an entire
year.

Why is this question important? The aim of the expedition will be to


19

parameterise the atmospheric, geophysical, oceanographic and all other possible


Page
variables in the Arctic, and use it to more accurately forecast the changes in our
weather systems.

24. Which among the following statements is/are true regarding ‘Green
Crackers’?

1. They are crackers with reduced emission levels.


2. The decibel level of crackers is the same as that of regular crackers.
3. The green crackers will be 25-30 per cent cheaper to manufacture.

Which of the statements given above is/are correct?

(a) 1 and 2 only


(b) 2 and 3 only
(c) 1 and 3 only
(d) 1, 2 and 3

Ans: (c) 1 and 3 only

Learning Zone: They are less harmful and less dangerous than conventional ones.
They are the crackers with reduced emission and decibel level. They are known as
‘green’ firecrackers because they have a chemical formulation that produces water
molecules, which substantially reduces emission levels and absorbs dust.

It promises a reduction in particulate matters and harmful gases, like nitrous oxide
and sulfur oxide, by 30- 35 per cent.
The green crackers will be 25-30 per cent cheaper to manufacture and
manufacturers would not have to make any changes in their facilities.

Why is this question important? Indian Council of Scientific and Industrial


Research (CSIR) has developed green crackers, which are new and improved
formulations of the previous sound-emitting crackers and other fireworks.

25. National e-Assessment Scheme (NeAC), recently initiated by the


government is for
20

(a) School Students


Page
(b) Central University Students
(c) Civil Servants performance analysis
(d) Income Tax Payers

Ans: (d) Income Tax Payers

Learning Zone: NeAC will be an independent office that will look after the work of
e-Assessment scheme which is recently notified for faceless e-assessment for
income taxpayers.

There would be a NeAC in Delhi to be headed by Principal Chief Commissioner of


Income Tax (Pr.CCIT).

There are 8 Regional e-Assessment Centres (ReAC) set up at Delhi, Mumbai,


Chennai, Kolkata Ahmedabad, Pune, Bengaluru and Hyderabad which would
comprise Assessment unit, Review unit, Technical unit and Verification units.

Each ReAC will be headed by Chief Commissioner of Income Tax (CCIT).


Cases for the specified work shall be assigned by the NeAC to different units by way
of automated allocation systems.

Why is this question important? The National e-Assessment Centre of IT


Department inaugurated recently.

26. With regard to eDantseva, recently seen in the news, consider the
following statements

1. It is launched by the Ministry of Health and Family Welfare.


2. It is the first-ever national digital platform on oral health information and
knowledge dissemination.
3. The website provides GPRS route and satellite images of the facility for easier
access to the general population.

Which of the statements given above is/are correct?


21

(a) 1 and 2 only


(b) 2 and 3 only
Page
(c) 1 and 3 only
(d) 1, 2 and 3

Ans: (d) 1, 2 and 3

Learning Zone: Launched recently by the Health and Family Welfare Ministry in
collaboration with AIIMS and other stakeholders.

e-DantSeva contains information about the National Oral Health Program, detailed
list of all the dental facility and colleges, Information, Education and Communication
(IEC) material and a unique feature called the ‘Symptom Checker’.

The website also provides GPRS route/images/satellite images of the facility for
easier access to the general population.

Why is this question important? It is the first-ever national digital platform on


oral health information and knowledge dissemination.

27. With reference to the Global Competitiveness Index, consider the


following statements

1. It is published by the World Bank.


2. In 2019 India moved down to 10 places compared with last years ranking.

Which of the statements given above is/are correct?

(a) 1 only
(b) 2 only
(c) Both 1 and 2
(d) Neither 1 nor 2

Ans: (b) 2 only

Learning Zone: Compared to last year, India has moved down 10 places to rank
68th. India was ranked 58th last year.
22
Page
It is among the worst-performing BRICS nations along with Brazil (ranked even
lower than India at 71st this year).

India ranks high in terms of macroeconomic stability and market size, while its
financial sector is relatively deep and stable despite the high delinquency rate, which
contributes to weakening the soundness of its banking system.

In innovation, India is well ahead of most emerging economies and on par with
several advanced economies.

Why is this question important? World economic forum has recently released
the Global competitiveness index

28. With reference to the Graded Response Action Plan (GRAP), often seen
in the news, consider the following statements

1. It works only as an emergency measure.


2. It includes plans and actions by various state governments to be taken
throughout the year to tackle pollution.
3. It is formulated by the Environment Pollution (Prevention and Control)
Authority.

Which of the statements given above is/are correct?

(a) 1 and 2 only


(b) 2 and 3 only
(c) 1 and 3 only
(d) 1, 2 and 3

Ans: (c) 1 and 3 only

Learning Zone: Measures aimed at stopping the use of diesel generator sets will
extend beyond Delhi to the NCR, where many areas see regular power cuts

It is approved by the Supreme Court in 2016. It works only as an emergency


23

measure.
Page
As such, the plan does not include action by various state governments to be taken
throughout the year to tackle industrial, vehicular and combustion emissions.

When the air quality shifts from poor to very poor, the measures listed have to be
followed since the plan is incremental in nature.

Why is this question important? Soon, some stricter measures to fight air
pollution will come into force in Delhi’s neighbourhood, as part of the Graded
Response Action Plan (GRAP).

29. With reference C40 initiative, often seen in the news consider the
following statements

1. It is a network of 40 countries around the world.


2. It is initiated by UNDP.
3. It aims to tackle climate change and driving urban action that reduces
greenhouse gas emissions and climate risks.

Which of the statements given above is/are correct?

(a) 1 only
(b) 3 only
(c) 2 and 3 only
(d) 1, 2 and 3

Ans: (b) 3 only


Learning Zone: C40 is a network of the world’s megacities committed to
addressing climate change.

The C40 Cities Climate Leadership Group (C40) is a group of 94 cities around the
world.

C40 is focused on tackling climate change and driving urban action that reduces
greenhouse gas emissions and climate risks while increasing the health, wellbeing
and economic opportunities of urban citizens.
24
Page
While C40 originally targeted megacities for their greater capacity to address climate
change, C40 now offers three types of membership categories to reflect the diversity
of cities taking action to address climate change, i.e, Megacities, Innovator cities and
Observer cities.

Why is this question important? C40 Cities World Mayors Summit is being held
in Copenhagen recently.

30. ‘The Triple Billion Target’ often seen in the news is an initiative of
which among the following?

(a) World Economic Forum (WEF)


(b) UNDP
(c) World Health Organization (WHO)
(d) UNEP

Ans: (c) World Health Organization (WHO)

Learning Zone: The WHO has embarked on its new 5-year strategic plan, with an
ambitious triple billion target: Ensuring 1 billion more people benefit from universal
health coverage, 1 billion more are protected from health emergencies and 1 billion
improve their overall health.

Why is this question important? The WHO India Country Cooperation Strategy
2019–2023: A Time of Transition’ has been launched.
The India CCS is one of the first that fully aligns itself with the newly adopted WHO
13th General Programme of Work and its ‘triple billion’ targets, the Sustainable
Development Goals (SDGs) and WHO South-East Asia Region’s eight Flagship
Priorities.

31. With reference to Pradhan Mantri Innovative Learning Programme


“DHRUV”, consider the following statements

1. It is a 5-year learning programme.


25

2. It is an initiative by ISRO.
Page
Which of the statements given above is/are correct?

(a) 1 only
(b) 2 only
(c) Both 1 and 2
(d) Neither 1 nor 2

Ans: (b) 2 only

Learning Zone: Pradhan Mantri Innovative Learning Programme- ‘DHRUV’, a 14-


day learning programme, was launched recently from Indian Space Research
Organisation (ISRO) Headquarters at Bengaluru.

The programme is called DHRUV (after the Pole Star) and every student to be called
‘DHRUV TARA’.

60 students, 30 each from Science and Performing Arts discipline, have been
selected for this ambitious programme in the first batch.

The programme begins with a tour at ISRO followed by a stay in Delhi, where the
selected students will be mentored by renowned experts.

Why is this question important? The program was launched recently from
Indian Space Research Organisation (ISRO) Headquarters at Bengaluru.

32. The Ministry of Women and Child Development (WCD) is developing a


POSHAN atlas in association with which among the following?

(a) Food and Agriculture Organization


(b) Bill & Melinda Gates Foundation
(c) NITI Aayog
(d) International Food Policy Research Institute

Ans: (b) Bill & Melinda Gates Foundation


26
Page
Learning Zone: The ministry of Women and Child Development (WCD) in
association with the Bill & Melinda Gates Foundation and Deendayal Research
Institute is developing a POSHAN atlas under POSHAN Abhiyan.

Why is this question important? It aims to map the crops and food grains grown
in different regions of the country so that nutritious protein-rich food in local areas
can be promoted.

27
Page
33. With reference to SUMAN initiative, recently seen in the news,
consider the following statements

1. It aims to provide dignified and quality health care at no cost to every woman
and newborn visiting a public health facility.
2. All pregnant women, newborns and mothers up to 6 months of delivery will
be able to avail the services offered by the initiative.

Which of the statements given above is/are correct?


(a) 1 only
(b) 2 only
(c) Both 1 and 2
(d) Neither 1 nor 2

Ans: (c) Both 1 and 2

Learning Zone: The Union Government has launched Surakshit Matritva


Aashwasan (SUMAN) to provide quality healthcare at zero cost to pregnant women,
new mothers and newborns.

It aims to provide dignified and quality health care at no cost to every woman and
newborn visiting a public health facility.

Under the scheme, the beneficiaries visiting public health facilities are entitled to
several free services.

These include at least four antenatal check-ups that also includes one checkup
during the 1st trimester, at least one checkup under Pradhan Mantri Surakshit
Matritva Abhiyan, Iron Folic Acid supplementation, Tetanus diphtheria injection.

The scheme aims to bring down the maternal and infant mortality rates in the nation
and to stop all preventable maternal and newborn deaths.

The scheme provides a positive and stress-free birth experience to the mother and
newborn.
28
Page
Why is this question important? The Union Government has launched Surakshit
Matritva Aashwasan (SUMAN) to provide quality healthcare at zero cost to pregnant
women, new mothers and newborns.

34. With reference to the Hindu Kush-Himalayan region, consider the


following statements

1. It spans an area of more than 4.3 million square kilometres in Afghanistan,


Pakistan, India and China only.
2. The region is popularly known as ‘The Third Pole’.
3. It is endowed with rich natural resources and contains all or part of four
global biodiversity hotspots.

Which of the statements given above is/are correct?

(a) 1 and 2 only


(b) 2 and 3 only
(c) 1 and 3 only
(d) 1, 2 and 3

Ans: (b) 2 and 3 only

Learning Zone: The Hindu Kush-Himalayan region spans an area of more than 4.3
million square kilometres in Afghanistan, Bangladesh, Bhutan, China, India,
Myanmar, Nepal, and Pakistan.

The region stores more snow and ice than anywhere else in the world outside the
polar regions, giving its name: ’The Third Pole‘.

It is endowed with rich natural resources and contains all or part of four global
biodiversity hotspots.

The mountain resources provide a wide range of ecosystem services and the basis
for the livelihoods to the 210 million people living in the region, as well as indirectly
to the 1.3 billion people — one-fifth of the worlds’ population — living in the
29

downstream river basins.


Page
Why is this question important? To better gauge the impact of climate change
on the Hindu Kush mountains, which includes the Himalayas, and spruce up data-
gathering, the India Meteorological Department (IMD) will collaborate with
meteorological agencies in China and Pakistan, among others, to provide climate
forecast services to countries in the region.

35. With reference to the proposed ‘Green wall’ of India, consider the
following statements

1. It is an initiative of planting trees along the coastline of India.


2. It is an initiative which helps in achieving the aim of India to restore 26
million hectares of degraded land by 2030.

Which of the statements given above is/are correct?


(a) 1 only
(b) 2 only
(c) Both 1 and 2
(d) Neither 1 nor 2

Ans: (b) 2 only

Learning Zone: It will be a 1,400km long and 5km wide green belt from Gujarat to
the Delhi-Haryana border, on the lines of the “Great Green Wall” running through
the width of Africa, from Dakar (Senegal) to Djibouti, to combat climate change and
desertification.

If approved, this may turn out to be a legacy programme in India’s efforts to deal
with land degradation and the eastward march of the Thar desert.

Why is this question important? India seeks to replicate the idea as a national
priority under its goal to restore 26 million hectares of degraded land by 2030.

36. Phazolicin, recently seen in the news is related to which among the
following?
30

(a) An antibiotic
(b) Space Telescope
Page
(c) Vaccine for Nipah Virus
(d) Pesticide

Ans: (a) An antibiotic

Learning Zone: It is a new antibiotic discovered in the soil of a tropical rainforest.


It was recently isolated deep in the tropical forests of Los Tuxtlas, Mexico.

Found in the root nodules of wild beans (Phaseolus vulgaris), this unusual antibiotic
is produced by a symbiotic soil bacterium that fixes nitrogen for the plant and keeps
harmful microbes away.

The antibiotic phazolicin is a class of peptide produced in the ribosome and is part of
a diverse class of natural products with a variety of biological uses.

Why is this question important? Not only could this antibiotic attack a diverse
group of bacterial cells, but the authors found it could also enter the bacteria and
bind to its ribosomes, messing with its ability to synthesise proteins – only the
second peptide is known to do this.

37. With reference to Election Manifesto, consider the following


statements

1. It shall not contain anything repugnant to the ideals and principles enshrined
in the Constitution.
2. It shall not contain a promise of such welfare measures on the basis of the
Directive Principles of State Policy enshrined in the Constitution.
3. They are not legally enforceable documents.

Which of the statements given above is/are correct?

(a) 1 and 3 only


(b) 2 and 3 only
(c) 1 and 2 only
(d) 1, 2 and 3
31

Ans: (a) 1 and 3 only


Page
Learning Zone: The election manifesto shall not contain anything repugnant to the
ideals and principles enshrined in the Constitution and further that it shall be
consistent with the letter and spirit of other provisions of Model Code of Conduct.

The Directive Principles of State Policy enshrined in the Constitution enjoin upon the
State to frame various welfare measures for the citizens and therefore there can be
no objection to the promise of such welfare measures in election manifestos.

They are a formality, and their release is often no more than a photo-op in the
campaign of political parties. They are frequently delayed, leaving no time for voters
to go through the contents in order to make informed choices.

Why is this question important? Election manifestos are not legally enforceable
documents. This has been reiterated by the Supreme Court in 2015.

38. NASA ICON Mission, recently seen in the news is related to which
among the following atmospheric layer?
(a) Exosphere
(b) Ionosphere
(c) Stratosphere
(d) Mesosphere

Ans: (b) Ionosphere

Learning Zone: National Aeronautics and Space Administration (NASA) has


launched a satellite ICON to detect dynamic zones of Earth’s Ionosphere. The
satellite Ionospheric Connection Explorer (ICON) was launched from an aircraft over
the Atlantic Ocean near the Florida coast.

The ICON mission is the 39th successful launch and satellite deployment by Pegasus
rocket.

This mission is operated by the University of California.


32
Page
Why is this question important? The ICON satellite will study the Earth’s
Ionosphere. It includes various layers of the uppermost atmosphere where free
electrons flow freely.

39. With reference to ‘Comprehensive Convention on International


Terrorism’ (CCIT), consider the following statements

1. It is an initiative by the Financial Action Task Force (FATF).


2. It intends to criminalize all forms of international terrorism and deny
terrorists, their financiers and supporters.
3. It is a draft proposed by India.

Which of the statements given above is/are correct?

(a) 1 and 3 only


(b) 2 and 3 only
(c) 2 only
(d) 1, 2 and 3

Ans: (b) 2 and 3 only

Learning Zone: The Comprehensive Convention on International Terrorism is a


proposed treaty which intends to criminalize all forms of international terrorism and
deny terrorists, their financiers and supporters access to funds, arms, and safe
havens. It is a draft proposed by India in 1996 that is yet to be adopted by the
UNGA.

Why is this question important? Vice President appeals to the world community
to ensure the early conclusion of the UN Comprehensive Convention on International
Terrorism.

40. With reference to Coalition for Disaster Resilient Infrastructure


(CDRI), consider the following statements
33

1. It is an international coalition of countries, the United Nations (UN) agencies,


multilateral development banks etc.
Page
2. It was first proposed by India in 2016.
3. It is launched in the 2019 UN Climate Action Summit.

Which of the statements given above is/are correct?

(a) 1 and 2 only


(b) 2 and 3 only
(c) 1 and 3 only
(d) 1, 2 and 3

Ans: (d) 1, 2 and 3

Learning Zone: The Coalition for Disaster Resilient Infrastructure (CDRI) is an


international coalition of countries, United Nations (UN) agencies, multilateral
development banks, the private sector, and academic institutions, that aims to
promote disaster-resilient infrastructure.

Its objective is to promote research and knowledge sharing in the fields of


infrastructure risk management, standards, financing, and recovery mechanisms. It
was launched by the Indian Prime Minister Narendra Modi at the 2019 UN Climate
Action Summit in September 2019.

Why is this question important? CDRI was first proposed by Indian Prime
Minister Narendra Modi during the 2016 Asian Ministerial Conference on Disaster
Risk Reduction held at Vigyan Bhavan, New Delhi.

41. With reference to the Asian Development Bank (ADB), consider the
following statements

1. It is dedicated to reducing poverty in Asia and the Pacific through inclusive


economic growth.
2. It provides loans as well as grants to the countries.
3. It is an observer to the United Nations.

Which of the statements given above is/are correct?


34

(a) 1 and 2 only


(b) 2 and 3 only
Page
(c) 1 and 3 only
(d) 1, 2 and 3

Ans: (d) 1, 2 and 3

Learning Zone: ADB defines itself as a social development organization that is


dedicated to reducing poverty in Asia and the Pacific through inclusive economic
growth, environmentally sustainable growth, and regional integration.

This is carried out through investments – in the form of loans, grants and
information sharing – in infrastructure, health care services, financial and public
administration systems, helping nations prepare for the impact of climate change or
better manage their natural resources, as well as other areas.

Why is this question important? GoI has signed a 190 million USD loan with the
Asian Development Bank (ADB) to upgrade road transport in the state of Rajasthan.

42. G20, The Global Smart Cities Alliance on Technology Governance is an


initiative of which among the following?

(a) UNDP
(b) World Economic Forum (WEF)
(c) Cities Climate Leadership Group
(d) World Bank

Ans: (b) World Economic Forum

Learning Zone: The Global Smart Cities Alliance’s founding set of institutional
partners include the presidents and host nations of the Group of 20 (G20) in 2019
and 2020; Japan and the Kingdom of Saudi Arabia; the Smart City Mission of India;
Cities for All; Cities Today Institute; Commonwealth Local Government Forum;
Commonwealth Sustainable Cities Network among others.

Why is this question important? India has joined the World Economic Forum’s
(WEF) G20 Global Smart Cities Alliance on Technology Governance.
35
Page
43. With reference to SARAS Aajeevika Mela, consider the following
statements
1. It is an initiative under Deendayal Antyodaya Yojana – National Rural
Livelihoods Mission (DAY – NRLM).
2. The objective of the initiative is to bring the rural businessmen under one
platform.

Which of the statements given above is/are correct?


(a) 1 only
(b) 2 only
(c) Both 1 and 2
(d) Neither 1 nor 2

Ans: (a) 1 only

Learning Zone: It is an initiative under Deendayal Antyodaya Yojana – National


Rural Livelihoods Mission (DAY – NRLM).

The objective of the initiative is to bring the rural women SHGs – Self Help Groups
under one platform.

Why is this question important? This is to help them showcase their skills, sell
their products and build linkages with buyers. Through the initiative, the women will
get exposure to understand the demand of urban customers.

44. With reference to National Medical Commission, Consider the


following statements

1. It replaces the Medical Council of India (MCI).


2. The members of the commission will be appointed by the central government
on the recommendation of a committee.
3. The chairperson of the commission is Minister of Health and Family Welfare
ministry.

Which of the statements given above is/are correct?


36

(a) 1 and 2 only


Page
(b) 2 and 3 only
(c) 1 and 3 only
(d) 1, 2 and 3

Ans: (a) 1 and 2 only

Learning Zone: National Medical Commission Act 2019 proposes to set up a


National Medical Commission with 25 members.

These members will be appointed by the central government on the


recommendation of a committee.

The members will include a chairperson, who must be a senior medical practitioner
and academic with at least 20 years of experience, 10 ex officio members and 14
part-time members.

Why is this question important? Government selects 25 members of National


Medical Commission.

For selecting members of the commission, that replaces the Medical Council of India
(MCI), the health ministry held a draw of lots.

The draw of the lot was held from the nominations sent by the state governments,
union territories and state medical councils.

The members include 10 Vice-Chancellors (VCs) from different states, nine members
from the State Medical Council (SMCs) and four part-time members from the
autonomous boards.

45. Which among the following lists are associated with the Financial
Action Task Force (FATF)?

1. Grey List
2. Black List
3. Dark Grey List
37

4. Red List
Page
Select the correct answer using the codes given below:

(a) 1 and 3 only


(b) 2, 3 and 4 only
(c) 1, 2 and 3 only
(d) 1 and 4 only

Ans: (c) 1, 2 and 3 only

Learning Zone: e FATF maintains the current list of nations: FATF blacklist
(formally called the "Call for action") and the "FATF greylist" (formally called the
'"Other monitored jurisdiction. According to FATF rules, there is one essential stage
between ‘Grey’ and ‘Black’ lists, referred to as ‘Dark Grey’.

‘Dark Grey’ means the issuance of a strong warning so that the country concerned
gets one last chance to improve, another official said.

Why is this question important? Pakistan is on the verge of strong action by the
international terror financing watchdog FATF and the country may be put in the
‘Dark Grey’ list, the last warning to improve.

46. Which among the following is/are Non - Tariff Measures in the
International Trade?

1. Sanitary And Phytosanitary Measures


2. Pre-Shipment Inspection
3. Customs Duty
4. Trade-Related Investment Measures

Select the correct answer using the codes given below:

(a) 1 and 2 only


(b) 2, 3 and 4 only
(c) 1, 3 and 4 only
(d) 1, 2 and 4 only
38

(e)
Ans: (d) 1, 2 and 4 only
Page
Learning Zone:

Technical measures
● Sanitary And Phytosanitary Measures
● Technical Barriers To Trade
● Pre-Shipment Inspection And Other Formalities
Non-technical measures
● Contingent Trade-Protective Measures
● Non-Automatic Licensing, Quotas, Prohibitions And Quantity-Control Measures
Other Than For SPS Or TBT Reasons
● Price-Control Measures, Including Additional Taxes And Charges
● Finance Measures
● Measures Affecting Competition
● Trade-Related Investment Measures
● Distribution Restrictions
● Restrictions On Post-Sales Services
● Subsidies (Excluding Export Subsidies Under P7)
● Government Procurement Restrictions
● Intellectual Property
● Rules Of Origin

Why is this question important? The Asia-Pacific Trade and Investment Report
2019 was published by the United Nations Economic and Social Commission for Asia
and the Pacific (ESCAP) and the United Nations Conference on Trade and
Development (UNCTAD).

Non-tariff measures (NTMs) have increased in the past two decades and are
affecting trade as well as sustainable development goals (SDGs) in Asian countries.

47. With reference to Lithium-Ion batteries, consider the following


statements

1. The energy density of lithium-ion is typically twice that of the standard nickel-
cadmium.
39

2. It has got a low self-discharge.


Page
Which of the statements given above is/are correct?

(a) 1 only
(b) 2 only
(c) Both 1 and 2
(d) Neither 1 nor 2

Ans: (c) Both 1 and 2

Learning Zone: A lithium-ion battery or Li-ion battery (abbreviated as LIB) is a


type of rechargeable battery. Lithium-ion batteries are commonly used for portable
electronics and electric vehicles and are growing in popularity for military and
aerospace applications.

The batteries have a high energy density, no memory effect (other than LFP cells)
and low self-discharge.

Why is this question important? John B. Goodenough, M. Stanley Whittingham


and Akira Yoshino have won the Nobel Prize in Chemistry 2019 “for the development
of lithium-ion batteries”.

48. Libra, a cryptocurrency recently launched by which among the


following?

(a) Facebook
(b) Google
(c) Finland
(d) Sweden

Ans: (a) Facebook

Learning Zone: Despite several high-profile defections and intense criticism from
US regulators and politicians, Facebook officially launches Libra.
40

The Libra Association, a nonprofit that will govern the currency, officially signed on
21 charter members at the organization’s inaugural meeting in Geneva.
Page
Why is this question important? Facebook says Libra is a “global currency and
financial infrastructure”. In other words, it is a digital asset built by Facebook and
powered by a new Facebook-created version of the blockchain, the encrypted
technology used by bitcoin and other cryptocurrencies.

49. With regard to LOTUS-HR project, consider the following statements

1. It is a project to manage plastic waste.


2. It is jointly supported by the Department of Biotechnology, Ministry of Science
and Technology and the Netherlands Organization for Scientific Research.

Which of the statements given above is/are correct?

(a) 1 only
(b) 2 only
(c) Both 1 and 2
(d) Neither 1 nor 2

Ans: (b) 2 only

Learning Zone: The LOTUS-HR project is jointly supported by the Department of


Biotechnology, Ministry of Science and Technology, Government of India and the
Netherlands Organization for Scientific Research.

The project was initiated in July 2017 and aims to demonstrate a novel holistic
(waste) water management approach that will produce clean water which can be
reused for various purposes.

Why is this question important? The launch of the second phase of the Local
Treatment of Urban Sewage Streams for Healthy Reuse (LOTUS-HR) program was
recently held. It is located in Delhi.

50. With reference to the Global Hunger Index, Consider the following
statements
41

1. It is published by the Food and Agriculture Organization.


Page
2. It is calculated on the basis of insufficient caloric intake, child undernutrition,
and child mortality.
3. India ranked 102 in 2019 report.

Which of the statements given above is/are correct?

(a) 1 and 2 only


(b) 2 and 3 only
(c) 1 and 3 only
(d) 1, 2 and 3

Ans: (b) 2 and 3 only

Learning Zone: The report is a peer-reviewed publication released annually by


Welthungerhilfe and Concern Worldwide.

The GHI scores are based on a formula that captures three dimensions of hunger:
insufficient caloric intake, child undernutrition, and child mortality—using four
component indicators

Why is this question important? India ranked 102 on the index among 117
qualifying countries with a score of 30.3. Even North Korea, Niger, Cameroon fared
better than India.

Neighbouring countries too bagged better spots — Sri Lanka (66), Nepal (73),
Pakistan (94) and Bangladesh (88).

51. Kurds, the world’s largest stateless ethnic group, belong to which
among the following countries?

1. Turkey
2. Iraq
3. Syria
4. Iran
5. Armenia
42

Select the correct answer using the codes given below:


Page
(a) 2, 4, and 5 only
(b) 1, 2, 3 and 4 only
(c) 1, 3, 4and 5 only
(d) 1, 2, 3, 4 and 5

Ans: (d) 1, 2, 3, 4 and 5

Learning Zone: The majority among the Kurdish people today are Sunni Muslim,
but there are adherents of other faiths too, including Sufism and other mystical
practices.

They live in the highlands of southern and eastern Turkey, northern Iraq,
northeastern Syria, northwestern Iran, and parts of south Armenia, and are a
minority in each of these countries. Small communities live in Georgia, Kazakhstan,
Lebanon, and eastern Iran as well.

Why is this question important? Recently, the Trump administration ordered US


troops to step aside from the border in northern Syria, effectively paving the way for
Turkey to launch an offensive against US-backed Kurdish forces who they regard as
enemies.

52. With regard to Microbial Fuel Cells (MFC), consider the following
statements

1. It is a device that converts chemical energy to electrical energy by the action


of microorganisms.
2. It can be used as a replacement for batteries.

Which of the statements given above is/are correct?

(a) 1 only
(b) 2 only
(c) Both 1 and 2
(d) Neither 1 nor 2
43

Ans: (a) 1 only


Page
Learning Zone: A microbial fuel cell (MFC) is a device that converts chemical
energy to electrical energy by the action of microorganisms.

MFCs are attractive for power generation applications that require only low power,
but where replacing batteries may be impractical, such as wireless sensor networks.
Wireless sensors, powered by microbial fuel cells can then, for example, be used for
remote monitoring (conservation).

Why is this question important? Microbial fuel cells have been installed at a zoo
in London. Using these cells, a plant has taken the botanical world’s first selfie.

53. With reference to One Nation One FASTag initiative recently seen in
the news, consider the following statements

1. FASTags are mandatory for all vehicles at all National Highways from
December 15.
2. Once FASTag is active drivers will no longer have to carry cash or stop to
make a transaction to pass the toll booths.

Which of the statements given above is/are correct?

(a) 1 only
(b) 2 only
(c) Both 1 and 2
(d) Neither 1 nor 2

Ans: (c) Both 1 and 2

Learning Zone: The government has already announced that FASTags will be
mandatory for all vehicles at all National Highways from December 15.

According to the National Highways Authority of India (NHAI), these devices will
make passing through tolls considerably smoother since drivers will no longer have
to carry cash or stop to make a transaction.
44
Page
Cameras at toll booths will take photos of passengers in a vehicle, which will be
useful for the Ministry of Home Affairs as there will be a record of a vehicle’s
movement.

Why is this question important? The scheme will be implemented from


December 15, 2019, and can be availed upon activation by new cars having Radio
Frequency Identification (RFID) tags on national and state highways throughout the
country.

The plan aims to integrate the collection of toll digitally and ensure seamless
mobility of vehicles across India.

54. With regard to Van Dhan Internship Programme, recently seen in the
news, consider the following statements

1. It is organised by TRIFED under the Ministry of Tribal Affairs.


2. It is a programme initiated by school students.

Which of the statements given above is/are correct?

(a) 1 only
(b) 2 only
(c) Both 1 and 2
(d) Neither 1 nor 2

Ans: (a) 1 only

Learning Zone: It is organised by TRIFED under the Ministry of Tribal Affairs.


18 interns (to be called Minister’s interns) from some of the reputed Institutes of
Rural Management/ Management Institutions/ Institutes of Social Work/ Social
Services of the country are participating.

Why is this question important? These Interns will help the tribal population in
becoming self-reliant and entrepreneurs.
45
Page
They will support the TRIFED activities on livelihood promotion, value addition of
Non-Timber Forest Products (NTFTs), marketing and credit linkages.

55. With regard to Food Safety Mitra (FSM) scheme, consider the following
statements

1. It is a scheme to support small and medium-scale food businesses to comply


46

with food safety laws.


Page
2. Food Safety Mitras will be chosen and they undergo training and certification
by FSSAI to do their work.
3. The FSM is a non-paid service.

Which of the statements given above is/are correct?

(a) 1 and 2 only


(b) 1 and 3 only
(c) 2 and 3 only
(d) 1, 2 and 3

Ans: (a) 1 and 2 only

Learning Zone: It is a scheme to support small and medium-scale food businesses


to comply with the food safety laws and facilitate licensing and registration, hygiene
ratings and training.

Why is this question important? Food Safety Mitras will be chosen. They
undergo training and certification by FSSAI to do their work and get paid by food
businesses for their services.

56. The 6X6X6 strategy, often seen in the news is related to which among
the following?

(a) Border management


(b) Gene Pool
(c) Anemia treatment
(d) Malaria treatment

Ans: (c) Anemia treatment

Learning Zone: The 6X6X6 strategy (six target beneficiary groups, six interventions
and six institutional mechanisms) of the programme has been highlighted for using
anaemia testing and treatment as the entry point to provide information on healthy
47

diets.
Page
Why is this question important? UNICEF released its State of the World’s
Children report for 2019.

The report analyses the global state of children’s health vis-a-vis malnutrition,
obesity, anaemia and other health issues.

57. Project Soli, often seen in the news is related to which among the
following?

(a) Initiative to increase soil fertility


(b) Sun exploration mission of Russia
(c) Free internet service by Facebook
(d) a radar-based chip to introduce Motion Sense by Google

Ans: (d) a radar-based chip to introduce Motion Sense by Google

Learning Zone: Recently launched Google Pixel 4 uses a radar-based Soli chip to
introduce Motion Sense, a feature that provides similar touchless gesture-based
controls.

Google announced Project Soli in 2015. Since then, Google’s ATAP (Advanced
Technology and Projects) division has been developing the technology, which can be
used in wearables, phones, computers, cars and IoT devices.

Why is this question important? The Soli radar chip works on the 60 GHz
spectrum frequency as it has the least interference for the kind of minute
movements Google wants to track. However, the 60 GHz spectrum is not
commercially usable in India.

The 60 GHz band is also known as V-band or WiGig band (Wi-Fi at 60 GHz) using
IEEE 802.11ad protocol.

58. With reference to the GOAL initiative, the Digital Skill Training
Programme for Tribal Women, consider the following statements
48

1. It is an initiative jointly launched by the Ministry of Tribal Affairs and


Facebook.
Page
2. It gives financial assistance to tribal women to buy mobile phones.

Which of the statements given above is/are correct?

(a) 1 only
(b) 2 only
(c) Both 1 and 2
(d) Neither 1 nor 2

Ans: (a) 1 only

Learning Zone: It is Jointly launched by the Ministry of Tribal Affairs and Facebook.
It aims at encouraging, inspiring and guiding tribal women from across India to
become local level digitally literate leaders.

The program will give technical support to the socially and economically marginalized
women for their needs to succeed, using the technology they may otherwise have
not had access to.

Why is this question important? 2nd phase of GOAL: Digital Skill Training
Programme for Tribal Women has recently started.

59. Which among the following statements is/are true regarding


Skandagupta Vikramaditya of Gupta dynasty

1. He was popularly known as the Saviour of India.


2. He successfully fought the battle against the invading Huns.
3. He appointed governors of all provinces.

Select the correct answer using the codes given below:

(a) 1 and 2 only


(b) 2 and 3 only
(c) 1 and 3 only
(d) 1, 2 and 3
49

Ans: (d) 1, 2 and 3


Page
Learning Zone: The Gupta period is known as the golden period of ancient Indian
history and Skandagupta Vikramaditya is known as the Saviour of India as he
successfully fought the battle against the invading Huns.

He ascended the throne in 455 AD and ruled till 467 AD. During the 12 year rule, he
not only defended the great culture of India but also from external aggression and
defeated the Huns (Bhitari pillar inscription), who had invaded India from the
northwest.

By defeating Pushyamitras during his initial years of ascendancy, Skandagupta


proved his ability to rule and took upon himself the title of Vikramaditya.

The Junagadh rock, which contains an inscription of the earlier rulers Ashoka and
Rudradaman, has an inscription engraved on the orders of Skandagupta’s governor
Parnadatta.

The inscription states that Skandagupta appointed governors of all provinces,


including Parnadatta as the governor of Saurashtra.

Why is this question important? Union home minister Amit Shah recently
inaugurated an international seminar on the role of Gupta dynasty emperor
Skandagupta at Banaras Hindu University in Varanasi. Extolling the greatness of
Skandagupta, Shah said history has been unfair to Gupta period rulers.

60. With regard to the Global Tuberculosis (TB) Report, consider the
following statements

1. It is released by WHO.
2. The tuberculosis incidence rate in India has decreased by almost 50,000
patients over the past year.

Which of the statements given above is/are correct?

(a) 1 only
50

(b) 2 only
(c) Both 1 and 2
Page
(d) Neither 1 nor 2

Ans: (c) Both 1 and 2

Learning Zone: The report provides a comprehensive and up-to-date assessment


of the TB epidemic and progress in the response at global, regional and country
levels for India.

The tuberculosis incidence rate in India has decreased by almost 50,000 patients
over the past year (26.9 lakh TB patients in India in 2018).

Incidence per 1,00,000 population has decreased from 204 in 2017 to 199 in 2018.
The number of patients being tested for rifampicin resistance has increased from
32% in 2017 to 46% in 2018.

Why is this question important? TB remains the top infectious killer in the world
claiming over 4,000 lives a day.

However, more people received life-saving treatment in 2018 than ever before,
largely due to improved detection and diagnosis.

61. As per the India Innovation Index 2019, Which among the following is
the most innovative major state in India?

(a) Karnataka
(b) Kerala
(c) Telangana
(d) Bihar

Ans: (a) Karnataka

Learning Zone: Karnataka is the most innovative major state in India.

Others in the top 10: Tamil Nadu, Maharashtra, Telangana, Haryana, Kerala, Uttar
Pradesh, West Bengal, Gujarat, and Andhra Pradesh.
51

The top ten major states are majorly concentrated in southern and western India.
Page
Sikkim and Delhi take the top spots among the northeastern & hill states, and union
territories/city-states/small states respectively.

Why is this question important? NITI Aayog with Institute for Competitiveness
as the knowledge partner has released the India Innovation Index (III) 2019.

62. With reference to Guru Ravidas, consider the following statements

1. He was a poet and saint of the Sufi Movement.


2. His devotional songs were included in the Sikh scriptures, Guru Granth Sahib.

Which of the statements given above is/are correct?

(a) 1 only
(b) 2 only
(c) Both 1 and 2
(d) Neither 1 nor 2

Ans: (b) 2 only

Learning Zone: He was a poet and saint of the Bhakti movement.


Born in Varanasi to the ‘untouchable’ leather tanners caste. He is believed to be a
disciple of the bhakti saint-poet Ramananand a contemporary of the bhakti saint-
poet Kabir. One of his famous disciples was the saint, Mirabai. Ravidas’ devotional
songs were included in the Sikh scriptures, Guru Granth Sahib.

Why is this question important? Yielding to the protests against the Supreme
Court-directed demolition of a Guru Ravidas temple in south Delhi, the Centre has
agreed to allow the same site to the devotees to construct a new temple.

63. With reference to talcum powder material, consider the following


statements

1. Talc is a mineral in clay mined from underground deposits.


2. It’s the softest mineral known to man and that makes it useful in a wide
52

range of consumer and industrial products.


Page
Which of the statements given above is/are correct?

(a) 1 only
(b) 2 only
(c) Both 1 and 2
(d) Neither 1 nor 2

Ans: (c) Both 1 and 2

Learning Zone: Talc is a mineral in clay mined from underground deposits. It’s the
softest mineral known to man and that makes it useful in a wide range of consumer
and industrial products.

Asbestos is also found underground, and veins of it can often be found in talc
deposits, leading to a risk of cross-contamination, geologists say.

Why is this question important? Johnson and Johnson (J&J) recalled 33,000
bottles of one lot of its talcum powder supplied in the United States (US) on October
18, 2019, as asbestos was found in it.

64. Aflatoxin M1 (AFM1), often seen in the news is related to:

(a) Toxins generally found in agricultural crops


(b) An antibiotic
(c) Bacteria that convert milk to curd
(d) Pesticide

Ans: (a) Toxins generally found in agricultural crops

Learning Zone: Aflatoxins are toxins produced by certain fungi which are generally
found in agricultural crops like maize, peanuts, cottonseed and others. They are
carcinogenic in nature, which means they can cause cancer.

Why is this question important? Consumption of food containing aflatoxin


53

concentrations of one milligram/kilogram or higher has been suspected to cause


Page
aflatoxicosis, the prognosis of which consists of acute liver failure, jaundice, lethargy
and nausea, eventually leading to death.

65. Danx- 19, recently seen in the news is a joint exercise by:

1. Indian Army
2. Indian Navy
54

3. Indian Air Force


4. Coast Guard
Page
Select the correct answer using the codes given below:

(a) 1, 2 and 3 only


(b) 1 and 3 only
(c) 2 and 4 only
(d) 1, 2, 3 and 4

Ans: (d) 1, 2, 3 and 4

Learning Zone: It is Carried out by the Indian Army, Navy, Air Force and Coast
Guard.
It is to validate the defensive plans of Headquarters ANC towards the pursuit of the
Command’s responsibility, namely ensuring the territorial integrity of the A&N
Islands.

Why is this question important? The second edition of Defence of Andaman &
Nicobar Islands 2019 (DANX-19), a large scale joint services exercise, was recently
conducted by Andaman and Nicobar Command (ANC).

66. With reference to Azad Hind Government, consider the following


statements

1. It was established in Singapore in 1915.


2. It was supported by the Axis powers.
3. Subash Chandra Bose was the head of the government.

Which of the statements given above is/are correct?

(a) 1 and 2 only


(b) 2 and 3 only
(c) 1 and 3 only
(d) 1, 2 and 3

Ans: (b) 2 and 3 only


55
Page
Learning Zone: Netaji Subhash Chandra Bose had announced the establishment of
the provisional government of Azad Hind in occupied Singapore in 1943.

Known as Arzi Hukumat-e-Azad Hind, it was supported by the Axis powers of


Imperial Japan, Nazi Germany, the Italian Social Republic, and their allies.

Bose was the head of the state, the prime minister and the minister for war and
foreign affairs.

Why is this question important? 76th anniversary of the formation of Azad Hind
Government on 21st October 2019.

67. State Chief Information Commissioner can be removed from the office
by the order of which among the following?

(a) President
(b) Governor
(c) Chief Minister
(d) Supreme Court of India

Ans: (b) Governor

Learning Zone: The state chief information commissioner is removed by the


Governor of the state under the RTI Act.

Why is this question important? Nine Karnataka information commissioners ask


the Governor to suspend their chief recently.

68. In the context of IMF quotas, quotas are distributed according to


which among the following parameters?

1. Country’s GDP
2. International Reserves
3. Economic openness
56

Select the correct answer using the codes given below:


Page
(a) 1 and 2 only
(b) 2 and 3 only
(c) 1 and 3 only
(d) 1, 2 and 3

Ans: (d) 1, 2 and 3

Learning Zone: IMF quotas are distributed according to a four-pronged formula


that considers a member country’s GDP, its economic openness, its “economic
variability” and international reserves.

Why is this question important? As per the latest deal, Members of the
International Monetary Fund (IMF) agreed to maintain its funding at $ 1 trillion but
postponed changes to its voting structure.

69. National Tiger Conservation Authority (NTCA) is created under which


among the following law?

(a) Environment Protection Act, 1986


(b) National Biodiversity Act, 2002
(c) Wildlife (Protection) Act, 1972
(d) Forest Conservation Act, 1980

Ans: (c) Wildlife (Protection) Act, 1972

Learning Zone: It is a statutory body under the Ministry of Environment, Forests


and Climate Change.
Constituted under enabling provisions of the Wildlife (Protection) Act, 1972, as
amended in 2006, for strengthening tiger conservation, as per powers and functions
assigned to it under the said Act.

Why is this question important? It is set up under the Chairmanship of the


Minister for Environment and Forests. The Authority will have eight experts or
professionals having qualifications and experience in wildlife conservation and
welfare of people including tribals, apart from three Members of Parliament of whom
57

two will be elected by the House of the People and one by the Council of States.
Page
70. For which among the following reason(s) lead is added to the paint?

1. Enhancing colour
2. Reducing Corrosion
3. Decreasing the drying time

Select the correct answer using the codes given below:

(a) 1 and 2 only


(b) 2 and 3 only
(c) 1 and 3 only
(d) 1, 2 and 3

Ans: (d) 1, 2 and 3

Learning Zone: Lead is added to paints for various reasons, including enhancing
the colour, reducing corrosion and decreasing the drying time.

However, lead can reach soil, dust and groundwater through weathering or peeling
of the patin.

Lead exposure accounted for 1.06 million deaths from long-term effects and 24.4
million disability-adjusted life years known as DALYs in 2007.

Lead can cause permanent damage to the brain and nervous system, resulting in
decreased IQ and increased behavioural problems

Why is this question important? World Lead Prevention Week started on


October 20, 2019.

On the sidelines, the United Nations Environment Programme (UNEP) has released a
report on lead concentration in items like paints.

71. With reference to ‘TechSagar’ recently seen in the news, consider the
following statements
58

1. It is a platform to discover India’s technological capability through a portal.


Page
2. It is launched by the Ministry of Human Resource Development

Which of the statements given above is/are correct?

(a) 1 only
(b) 2 only
(c) Both 1 and 2
(d) Neither 1 nor 2

Ans: (a) 1 only

Learning Zone: It is a platform to discover India’s technological capability through


a portal.

It is a consolidated and comprehensive repository of India’s cyber tech capabilities


which provides actionable insights about capabilities of the Indian Industry,
academia and research across 25 technology areas like internet of things (IoT),
Artificial Intelligence (AI), Machine Learning (ML), blockchain, cloud & virtualisation,
robotics & automation, AR/VR, wireless & networking, and more.

Why is this question important? It has been launched by the National Cyber
Security Coordinator’s office in partnership with the Data Security Council of
India(DSCI).

72. With reference to BHIM app, consider the following statements

1. It is a UPI based payment interface.


2. It is developed by the Reserve Bank of India.
3. It supports all languages in the 8th schedule of the constitution.

Which of the statements given above is/are correct?

(a) 1 only
(b) 1 and 2 only
(c) 2 and 3 only
59

(d) 1, 2 and 3
Page
Ans: (a) 1 only

Learning Zone: Bharat Interface for Money (BHIM) is a UPI based payment
interface, developed by National Payments Corporation of India (NPCI). It allows
real-time fund transfer.
It was Launched in December 2016.

73. In the context of National River Ganga (Rejuvenation, Conservation


and Management) Bill, 2018, consider the following statements

1. It proposes to ban the construction of jetties, ports or “permanent hydraulic


structures” in the Ganga unless permitted by the National Ganga
Rejuvenation Authority.
2. The Armed Ganga Protection Corps (GPC) personnel will be provided by the
ministry of home affairs for the conservation of the river.

Which of the statements given above is/are correct?

(a) 1 only
(b) 2 only
(c) Both 1 and 2
(d) Neither 1 nor 2

Ans: (c) Both 1 and 2

Learning Zone: The bill proposes to ban the construction of jetties, ports or
“permanent hydraulic structures” in the Ganga unless permitted by the National
Ganga Rejuvenation Authority.

It proposes to create a management structure that will supervise the health of the
2,500 kilometre-long Ganga which, the draft Bill defines, as ‘India’s national river.’

The Bill lays down a host of restrictions to ensure the “uninterrupted, ecological
flow” of the river. Currently, a host of dams in the upper stretches of the river lead
to the river’s flow being obstructed.
60
Page
The Armed Ganga Protection Corps (GPC)personnel will be provided by the ministry
of home affairs and will be deployed by the National Ganga Rejuvenation Authority.
The GPC personnel will have the power to arrest those who pollute the river
covering offences like obstructing the flow of the river to commercial fishing.

Why is this question important? In the next two months, hydropower projects
that do not comply with the Centre’s ecological flow notification, which mandates
that project developers ensure a minimum supply of water all through the year,
could face closure.

74. With regard to the Defence Acquisition Council (DAC), consider the
following statements

1. It is headed by the Prime Minister of India.


2. DAC is to ensure expeditious procurement of the approved requirements of
the Armed Forces.
3. It clears all acquisitions which include both imported and those produced
indigenously.

Which of the statements given above is/are correct?

(a) 1 and 2 only


(b) 2 and 3 only
(c) 1 and 3 only
(d) 1, 2 and 3

Ans: (b) 2 and 3 only

Learning Zone: The objective of the DAC is to ensure expeditious procurement of


the approved requirements of the Armed Forces, in terms of capabilities sought, and
time frame prescribed, by optimally utilizing the allocated budgetary resources.

The DAC is responsible for giving policy guidelines to acquisitions, based on long-
term procurement plans. It also clears all acquisitions, which includes both imported
and those produced indigenously or under a foreign license.
61
Page
Why is this question important? To counter corruption and speed up decision-
making in military procurement, the government of India in 2001 decided to set up
an integrated DAC. It is headed by the Defence Minister.

75. With reference to Defence Technologies and Trade Initiative (DTTI),


consider the following statements

1. It is to ensure expeditious procurement of the approved requirements of the


Armed Forces.
2. It is headed by Secretary for Defence Protection, India.

Which of the statements given above is/are correct?

(a) 1 only
(b) 2 only
(c) Both 1 and 2
(d) Neither 1 nor 2

Ans: (b) 2 only

Learning Zone: It came about to expedite scope of cooperation between partner


countries on defence technology that become narrow due to the presence of
different bureaucratic processes and legal requirements.

Essentially, DTII is an initiative to provide increased US senior-level oversight and


engagement to get beyond these obstacles.

Why is this question important? The DTTI initiative is led by Undersecretary of


Defence for Acquisition and Sustainment from US and Secretary for Defence
Protection from India.

The aim of DTTI is to bring sustained leadership focus to the bilateral defence trade
relationship and create opportunities for co-production and co-development of
defence equipment.
62

76. Which among the following area is the focus of ‘Intensified Mission
Indradhanush 2.0’?
Page
(a) North - East India
(b) Jammu & Kashmir
(c) Union Territories
(d) Bihar and UP

Ans: (d) Bihar and UP

Learning Zone: To ensure that not a single child in the country misses out on
vaccination, the government will launch the ‘Intensified Mission Indradhanush 2.0‘
on October 31 with a special focus on improving coverage in areas with “low”
immunisation.

Why is this question important? Through ‘IMI 2.0’, the health ministry aims to
reach each and every child below the age of two years and all pregnant women still
uncovered/partially covered in 271 districts of the country and 652 blocks of Uttar
Pradesh and Bihar.

77. Global Wealth Report 2019, recently released by which among the
following?

(a) The Credit Suisse Group


(b) World Economic Forum
(c) World Bank
(d) International Development Association

Ans: (a) The Credit Suisse Group

Learning Zone: The Credit Suisse Group, a Switzerland-based multinational


investment bank, has released the 10th edition of its annual Global Wealth Report.

Why is this question important? India remains one of the fastest wealth creators
in the world, with household wealth in dollar terms growing faster than any other
region.
63

Wealth per Indian adult is $14,569 ( ₹10.31 lakh as on 21 October). However, the
average number is skewed heavily by a few wealthy individuals.
Page
78. With reference to ‘Feed our future’ cinema ad campaign, consider the
following statements

1. It is launched by the United Nations World Food Programme (WFP).


2. It aims to raise awareness and take steps against hunger and malnutrition in
India.

Which of the statements given above is/are correct?

(a) 1 only
(b) 2 only
(c) Both 1 and 2
(d) Neither 1 nor 2

Ans: (c) Both 1 and 2

Learning Zone: The United Nations World Food Programme (WFP) have launched
a cinema advertisement campaign ‘Feed Our Future’.

Why is this question important? It aims to raise awareness and take steps
against hunger and malnutrition in India.

It is launched in collaboration with UFO Movies.

The ad shows the reality that millions of people facing across the world. The ad
shows that the world has to face great loss when children’s voices are silenced due
to hunger.

79. With regard to Thirty Meter Telescope (TMT), consider the following
statements

1. It is an international project being funded by scientific organisations of


Canada, China, India, Japan and the USA.
2. The planned location for the telescope is Andaman and Nicobar Islands.
64

Which of the statements given above is/are correct?


Page
(a) 1 only
(b) 2 only
(c) Both 1 and 2
(d) Neither 1 nor 2

Ans: (a) 1 only

Learning Zone: The Thirty Meter Telescope (TMT) is an astronomical observatory


with an extremely large telescope (ELT).

It is an international project being funded by scientific organisations of Canada,


China, India, Japan and the USA.

The Planned location is Mauna Kea on the island of Hawaii in the US state of
Hawaii.

The TMT is designed for near-ultraviolet to mid-infrared observations, featuring


adaptive optics to assist in correcting image blur.

Why is this question important? Protests have erupted in Hawaii against the
installation of TMT

80. With reference to Col Chewang Rinchen Setu, the recently built
bridge, consider the following statements

1. It is India’s highest altitude all-weather permanent bridge.


2. It is built on Jhelum river
3. The bridge is sandwiched between strategic Karakoram and Chang Chenmo
ranges.

Which of the statements given above is/are correct?

(a) 1 and 2 only


(b) 1 and 3 only
65

(c) 2 and 3 only


(d) 1, 2 and 3
Page
Ans: (b) 1 and 3 only

Learning Zone: Col Chewang Rinchen Setu is India’s highest altitude all-weather
permanent bridge.

It is located in eastern Ladakh at nearly 45 km from the country’s border with China.

Why is this question important? The bridge is strategically located on the 255-
km Darbuk-Shayok-Daulat Beg Oldie (DSDBO) section of the road between Leh and
Karakoram Pass. It is built on the Shyok River.

The bridge is sandwiched between strategic Karakoram and Chang Chenmo ranges.

81. With reference to Ozone Layer, consider the following statements

1. The ozone layer exists mainly in the stratosphere.


2. The Stratosphere is a layer of the atmosphere that reaches from 50 to 80
kilometres above the Earth’s surface.
3. The ozone hole is a region of depleted layers of ozone above the Antarctic
region.

Which of the statements given above is/are correct?

(a) 1 and 2 only


(b) 2 and 3 only
(c) 1 and 3 only
(d) 1, 2 and 3

Ans: (c) 1 and 3 only


Learning Zone: A layer of ozone envelops the Earth and keeps damaging
ultraviolet, or UV, radiation from reaching living things on the planet’s surface.

The ozone layer exists mainly in the stratosphere, a layer of the atmosphere that
reaches from 10 to 50 kilometres (about 6 to 30 miles) above the Earth’s surface.
66
Page
The ozone hole is a region of depleted layers of ozone above the Antarctic region,
whose creation is linked to increased cases of skin cancer.

Manufactured chemicals deplete the ozone layer. Each spring over Antarctica,
atmospheric ozone is destroyed by chemical processes. This creates the ozone hole,
which occurs because of special meteorological and chemical conditions that exist in
that region.

Why is this question important? During September and October, the annual
ozone hole over the Antarctic reached its peak extent of 16. 4 million sq km on
September 8, then shrank to less than 10 million sq km. This has been the smallest
observed since 1982. NASA has described it as great news for the Southern
Hemisphere.

82. Which among the following is/are facilities of Government


e-Marketplace (GeM)?

1. Buying Goods and Services online, as and when required.


2. Useful for low value buying and also for bulk buying at a competitive price
using Reverse Auction/ e-bidding.
3. Return policy.

Select the correct answer using the codes given below:

(a) 1 and 2 only


(b) 2 and 3 only
(c) 1 and 3 only
(d) 1, 2 and 3

Ans: (d) 1, 2 and 3

Learning Zone: GeM facilities are

● Listing of products for individual, prescribed categories of Goods/ Services of


common use.
67

● Look, estimate, compare and buy facilities on a dynamic pricing basis.


● Marketplace buying of a majority of common User Items.
Page
● Buying Goods and Services online, as and when required.
● Transparency and ease of buying.
● Useful for low value buying and also for bulk buying at a competitive price
using Reverse Auction/ e-bidding.
● Continuous vendor rating system.
● Return policy.

Why is this question important? The Government e-Marketplace (GeM) has


signed a memorandum of understanding (MoU) with Federal Bank to offer various
services.

The MoU will facilitate a cashless, paperless and transparent payment system on the
portal and would create an efficient procurement system for government entities.

83. With regard to Ease of Doing business report, consider the following
statements

1. It is published by the World Bank.


2. India achieved the target of being at 50th place this year.

Which of the statements given above is/are correct?

(a) 1 only
(b) 2 only
(c) Both 1 and 2
(d) Neither 1 nor 2

Ans: (a) 1 only

Learning Zone: It provides objective measures of business regulations and their


enforcement across 190 economies and selected cities at the subnational and
regional level.

India went up 14 rungs in the 2020 survey to score a 63, making it one of the
world’s top 10 most improved countries for the third consecutive time.
68
Page
However, India failed to achieve the government's target of being at 50th place. It
was 77th last year.

Why is this question important? This is the third year in a row that India has
made it to the top 10 in Doing Business, which is a success which very few countries
have done over the 20 years of the project.

84. With reference to Snow Leopard, Consider the following statements

1. It is a critically endangered species.


2. It is the state animal of Himachal Pradesh.

Which of the statements given above is/are correct?

(a) 1 only
(b) 2 only
(c) Both 1 and 2
(d) Neither 1 nor 2

Ans: (b) 2 only

Learning Zone: It is listed as Vulnerable on the IUCN Red List of Threatened


Species.

Inhabit alpine and subalpine zones at elevations from 3,000 to 4,500 m (9,800 to
14,800 ft).

It is State animal of Himachal Pradesh and the National Heritage Animal of Pakistan.

Why is this question important? First National Protocol to Enumerate Snow


Leopard Population in India has been launched on the occasion of International
Snow Leopard Day.

85. Kartarpur Sahib Corridor Agreement, recently seen in the news is


between India and which among the following?
69

(a) Afghanistan
Page
(b) Pakistan
(c) Bangladesh
(d) Myanmar

Ans: (b) Pakistan

Learning Zone: India signs the Kartarpur Sahib Corridor Agreement with Pakistan.
The Agreement lays down a formal framework for operationalisation of the Kartarpur
Sahib Corridor.

Why is this question important? Indian pilgrims of all faiths and people of Indian
origin can use the corridor. The trip will be Visa-Free. Pilgrims need to carry only a
valid passport. Persons of Indian Origin need to carry an OCI card along with the
passport of their country.

86. With regard to Kanya Sumangala Yojana, recently seen in the news,
consider the following statements?

1. It is to be launched for the girl child in UP.


2. It will provide a fund worth Rs 15000 to every family where a girl child is
born.

Which of the statements given above is/are correct?

(a) 1 only
(b) 2 only
(c) Both 1 and 2
(d) Neither 1 nor 2

Ans: (c) Both 1 and 2

Learning Zone: Kanya Sumangala Yojana to be launched for the girl child in UP.
It will provide a fund worth Rs 15000 to every family where a girl child is born. The
amount will be released to the family in a phased manner.
70
Page
The scheme has been designed in a way that the parents will have to take proper
care of the girl child with respect to her health and education and other aspects, in
order to get the benefit.

87. With regard to Sustainable Mobility for All (SuM4All) initiative,


consider the following statements
71

1. It is an initiative by the World Bank.


Page
2. It brings together public and private organisations and companies to act
collectively to implement the SDGs and transform the transport sector.
3. It released a Global mobility report.

Which of the statements given above is/are correct?

(a) 1 and 2 only


(b) 2 and 3 only
(c) 1 and 3 only
(d) 1, 2 and 3

Ans: (d) 1, 2 and 3

Learning Zone: Established in 2017 and hosted by the World Bank, SuM4All is a
global coalition of 55 influential public organizations and private companies with a
shared ambition to transform the future of mobility.

Why is this question important? Global Mobility Report has been released by
Sustainable Mobility for All (SuM4All) initiative recently.

88. With reference to Chenani-Nashri tunnel, consider the following


statements

1. It is on NH 44 in Jammu & Kashmir.


2. It is the longest such state of the art tunnel in the country.

Which of the statements given above is/are correct?

(a) 1 only
(b) 2 only
(c) Both 1 and 2
(d) Neither 1 nor 2

Ans: (c) Both 1 and 2


72

Learning Zone: The 9km Chenani Nashri Tunnel is on NH 44 in Jammu & Kashmir.
Page
It is the longest such state of the art tunnel in the country, connecting Udhampur to
Ramban in Jammu.

It is the world’s sixth road tunnel with a transverse ventilation system enabled and
controlled by ABB software.

Why is this question important? It is Renamed after Dr Syama Prasad


Mukherjee.

89. With reference to the Central Vigilance Commission, consider the


following statements

1. It is a statutory body.
2. It submits its report to the Parliament
3. It consists of a central vigilance commissioner along with 2 vigilance
commissioners.

Which of the statements given above is/are correct?

(a) 1 only
(b) 1 and 3 only
(c) 2 and 3 only
(d) 1, 2 and 3

Ans: (b) 1 and 3 only

Learning Zone: It is the apex vigilance institution created via executive resolution
(based on the recommendations of Santhanam committee) in 1964 but was
conferred with statutory status in 2003.

It submits its report to the President of India.

Why is this question important? Vigilance Awareness Week is observed from


28th October to 2nd November.
73

The theme of the Vigilance Awareness Week: “Integrity- A way of life”.


Page
90. With reference to developing countries in the WTO, consider the
following statements

1. There are no WTO definitions of “developed” and “developing” countries.


2. Members announce for themselves whether they are “developed” or
“developing” countries.
3. Other members cannot challenge the decision of a member to make use of
provisions available to developing countries.

Which of the statements given above is/are correct?

(a) 1 only
(b) 1, 2 and 3
(c) 2 and 3 only
(d) 1 and 2 only

Ans: (d) 1 and 2 only

Learning Zone: There are no WTO definitions of “developed” and “developing”


countries. Members announce for themselves whether they are “developed” or
“developing” countries.

However, other members can challenge the decision of a member to make use of
provisions available to developing countries.

Developing country status in the WTO brings certain rights.

Why is this question important? South Korea has said that it will no longer seek
special treatment reserved for developing countries by the World Trade Organization
in future negotiations given its enhanced global economic status.

91. With reference to IndiGen Genome project, consider the following


statements
74

1. The initiative is implemented by India and Switzerland.


Page
2. It aims to undertake the whole genome sequencing of thousands of
individuals representing diverse ethnic groups from India.
3. It will result in predictive and preventive medicine with a faster and efficient
diagnosis of rare genetic diseases.

Which of the statements given above is/are correct?

(a) 1 and 2 only


(b) 2 and 3 only
(c) 1 and 3 only
(d) 1, 2 and 3

Ans: (b) 2 and 3 only

Learning Zone: The outcomes of the IndiGen will have applications in a number of
areas including predictive and preventive medicine with a faster and efficient
diagnosis of rare genetic diseases.

The data will be important for building the knowhow, baseline data and indigenous
capacity in the emerging area of Precision Medicine.

Why is this question important? The initiative was implemented by the CSIR-
Institute of Genomics and Integrative Biology (IGIB), Delhi and CSIR-Centre for
Cellular and Molecular Biology (CCMB), Hyderabad.

92. “Colombo Declaration”, recently adopted by the UN Environment


Programme (UNEP) member states is related to which among the
following?

(a) Tackling air pollution due to stubble burning


(b) Conservation of polar regions
(c) Tackling the global nitrogen challenge.
(d) Vehicular pollution

Ans: (c) Tackling global nitrogen challenge.


75
Page
Learning Zone: The Colombo Declaration has been developed with the technical
support of the International Nitrogen Management System (INMS), a joint activity of
the UNEP and the International Nitrogen Initiative supported by the Global
Environmental Facility. The aim is to halve nitrogen waste by 2030.

Why is this question important? UN Environment Programme (UNEP) member


states recently adopted the “Colombo Declaration” which calls for tackling global
nitrogen challenge.

93. With reference to the Indian Ocean Rim Association (IORA), consider
the following statements

1. It consists of 22 coastal states bordering the Indian Ocean.


2. It was set up in 1997.
3. It is based on the principles of Open Regionalism for strengthening Economic
cooperation.

Which of the statements given above is/are correct?

(a) 1 and 2 only


(b) 2 and 3 only
(c) 1 and 3 only
(d) 1, 2 and 3

Ans: (d) 1, 2 and 3

Learning Zone: The Indian Ocean Rim Association was set up with the objective of
strengthening regional cooperation and sustainable development within the Indian
Ocean Region.

Setup in 1997, it Consists of 22 coastal states bordering the Indian Ocean.

The IORA is a regional forum, tripartite in nature, bringing together representatives


of Government, Business and Academia, for promoting co-operation and closer
interaction among them.
76
Page
It is based on the principles of Open Regionalism for strengthening Economic
Cooperation particularly on Trade Facilitation and Investment, Promotion as well as
Social Development of the region

Why is this question important? The 19th IORA Council of Ministers meeting is
held on November 7 in Abu Dhabi with the theme of “Promoting a Shared Destiny
and Path to Prosperity in the Indian Ocean”.

The meeting is important as two of India’s important partners, the United Arab
Emirates and Bangladesh, will take charge as the new chair and vice-chair of one of
the largest regional maritime organisations for the duration of 2019-21.

94. With reference to organoids, often seen in the news, consider the
following statements

1. They are a group of cells grown in laboratories into three-dimensional,


miniature structures.
2. It mimics the cell arrangement of a fully-grown organ.
Which of the statements given above is/are correct?

(a) 1 only
(b) 2 only
(c) Both 1 and 2
(d) Neither 1 nor 2

Ans: (c) Both 1 and 2

Learning Zone: Organoids are a group of cells grown in laboratories into three-
dimensional, miniature structures that mimic the cell arrangement of a fully-grown
organ.

They are tiny (typically the size of a pea) organ-like structures that do not achieve
all the functional maturity of human organs but often resemble the early stages of
developing tissue.
77

Most organoids contain only a subset of all the cells seen in a real organ but lack
blood vessels to make them fully functional.
Page
Why is this question important? We’ve heard a lot in the last few years about
organoids, the so-called “brains in a dish” created in labs by neuroscientists.

However, experts have expressed concerns over growing mini-brains or organoids in


the laboratory that can perceive or feel things.

95. Which among the following is an ‘absentee voter’ according to the


Election Commission of India?

(a) someone who is employed in essential services


(b) An ill voter who cannot present be present in the polling booth
(c) All voters who do not vote in the general election
(d) Ministers, MP’s and MLA’s

Ans: (a) someone who is employed in essential services

Learning Zone: An absentee voter is someone who is employed in “essential


services”. The EC will notify which jobs and professions are covered under “essential
services” after consulting the government.

A concept of ‘absentee voter’ has been introduced and defined for the elections.
People under the new category can choose to vote through postal ballot by filling up
Form 12D and submitting it to the nodal officer within five days of notification of an
election.

These votes will be registered at a special centre specified by the Election


Commission (EC).

Why is this question important? Election Commission of India has started


working on detailed guidelines and SoPs to facilitate the process of the postal ballot
paper for Absentee voters of essential services, Senior citizens of more than 80
years and marked PwD electors.

96. With reference to BASIC Countries, Consider the following statements


78
Page
1. It is formed as the result of an agreement signed by the four countries on
November 28, 2009.
2. The BASIC countries hold 40 per cent of the world's population.

Which of the statements given above is/are correct?

(a) 1 only
(b) 2 only
(c) Both 1 and 2
(d) Neither 1 nor 2

Ans: (c) Both 1 and 2

Learning Zone: The BASIC group was formed as the result of an agreement signed
by the four countries on November 28, 2009.

They are a bloc of four large newly industrialized countries – Brazil, South Africa,
India and China.

The signatory nations have a broadly common position on reducing greenhouse gas
emissions and raising the massive funds that are needed to fight climate change.

The BASIC countries constituted one of the parties in the Copenhagen Accord
reached with the US-led grouping; the Accord, was, however, not legally binding.

Why is this question important? The 29th BASIC Ministerial Meeting on Climate
Change was held recently in Beijing, China.

97. With regard to Universal Postal Union (UPU), consider the following
statements

1. It is a specialized agency of the United Nations.


2. It frames rules for international mail exchange and fixes rates for international
postal services.
3. Both India and Pakistan are members of UPU.
79

Which of the statements given above is/are correct?


Page
(a) 1 and 2 only
(b) 2 and 3 only
(c) 1 and 3 only
(d) 1, 2 and 3

Ans: (d) 1, 2 and 3


Learning Zone: It is a specialized agency of the United Nations that coordinates
postal policies among member nations, in addition to the worldwide postal system.

Established in 1874 and is the second oldest international organization worldwide


after the International Telecommunication Union (ITU) which was established in
1865.

Headquarter is in Berne, Switzerland. It has 192 member countries.


India joined the UPU on July 1, 1876, and Pakistan on November 10, 1947.

Why is this question important? In a unilateral decision, Pakistan has stopped


the exchange of postal mails with India since August 27.

98. With regard to the Copernicus programme, often seen in the news,
consider the following statements

1. It is a moon observation programme.


2. This initiative is headed by the European Commission (EC) in partnership with
the European Space Agency (ESA).
3. ESA is developing a new family of satellites, called Sentinels, specifically for
the operational needs of the Copernicus programme.

Which of the statements given above is/are correct?

(a) 1 and 3 only


(b) 1 and 2 only
(c) 2 and 3 only
(d) 1, 2 and 3
80

Ans: (c) 2 and 3 only


Page
Learning Zone: Copernicus is the most ambitious Earth observation programme to
date.

It will provide accurate, timely and easily accessible information to improve the
management of the environment, understand and mitigate the effects of climate
change and ensure civil security.

Copernicus is the new name for the Global Monitoring for Environment and Security
programme, previously known as GMES.

This initiative is headed by the European Commission (EC) in partnership with the
European Space Agency (ESA).

Why is this question important? Data from the Sentinel-3 World Fire Atlas have
been released. Data have been recorded under the Copernicus Sentinel-3 mission.

99. With reference to National Digital Health Blueprint, consider the


following statements

1. It aims to deploy Artificial Intelligence (AI) in leveraging health records.


2. The blueprint proposes a National Digital Health Mission “as a pure
government organisation.

Which of the statements given above is/are correct?

(a) 1 only
(b) 2 only
(c) Both 1 and 2
(d) Neither 1 nor 2

Ans: (c) Both 1 and 2

Learning Zone: It lays out the ‘building blocks’ for the implementation of the
National Health Stack (NHS), which aims to deploy Artificial Intelligence (AI) in
81

leveraging health records.


Page
Keeping true to the government’s larger agenda, of ‘data as a public good’, the
blueprint proposes the linking of multiple databases to generate greater and
granular data that can be leveraged by the public as well as the private sector –
including insurance companies, hospitals, apps and researchers.

The blueprint proposes a National Digital Health Mission “as a purely government
organisation with complete functional autonomy adopting some features of some of
the existing National Information Utilities like UIDAI and GSTN.”

Why is this question important? National Digital Health Blueprint report


submitted to the Health Ministry.

100. With reference to Edge Computing, consider the following statements

1. In edge computing, the data is analysed remotely in different locations.


2. It allows for quicker data processing and content delivery.

Which of the statements given above is/are correct?

(a) 1 only
(b) 2 only
(c) Both 1 and 2
(d) Neither 1 nor 2

Ans: (b) 2 only

Learning Zone: Edge computing enables data to be analysed, processed, and


transferred at the edge of a network. Meaning, the data is analysed locally, closer to
where it is stored, in real-time without latency, rather than sending it far away to a
centralised data centre.

It allows for quicker data processing and content delivery.

The basic difference between edge computing and cloud computing lies in where the
data processing takes place.
82
Page
Why is this question important? According to research, By 2025, companies will
generate and process more than 75% of their data outside of traditional centralised
data centres — that is, at the “edge” of the cloud.

83
Page
ClearIAS FREE Online Study Materials
Click on the below links to read/download the online notes in ClearIAS.

ClearIAS Study Materials – Useful for UPSC Prelims and Mains

• Indian History Notes – Indian History and Culture.


• Geography Notes – Indian and World Geography.
• Economics Notes – Indian Economy.
• Indian Polity Notes – Constitution, Social Justice etc.
• General Science Notes – Science and Technology.
• Environment Notes – Biodiversity, Climate Change etc.
• Current Affairs Notes – National and international.

ClearIAS Study Materials – For Additional areas in UPSC Mains

• World History Notes – GS paper 1.


• Indian Society Notes – GS paper 1.
• Foreign Relations Notes – GS paper 2.
• International Affairs Notes – GS paper 2.
• Internal Security Notes – GS paper 3.
• Disaster Management Notes – GS paper 3.
• Ethics, Integrity and Aptitude Notes – GS paper 4.

ClearIAS UPSC Mains Special: Paper-wise Study Materials

• Essay Paper
• GS 1 Paper
• GS 2 Paper
• GS 3 Paper
• GS 4 Paper
Road Map to
CLEAR IAS
START

2 1

Read NCERT Text Join ClearIAS


Books from Class Prelims Online Mock
6-12. Test Series

3 4

Learn ClearIAS Notes Read the standard


(Easy-to-Learn reference text books
Online Study Materials) (clearias.com/ias-books)

6 5

Take ClearIAS Mains Finish your optional subject


Mock Exams and and GS Topics as per
polish your answer ClearIAS Prelims cum Mains
writing skills Integrated approach.

Get your bio-data Success!


reviewed by ClearIAS
Experts.Take ClearIAS
Mock Interviews.

Download ClearIAS App; Clear IAS by Self-Study!


www.clearias.com
The ClearIAS 'TTT' Approach
to clear the IAS exam

Text-books Test-series Techniques

Success in UPSC CSE is:


(1) 50% Knowledge (Text-books)
(2) 50% Skills (Techniques)
PS: ClearIAS Test Series combines both.
The ClearIAS Approach:
Knowledge + Skills

Learn NCERTS and


Core Areas Standard Text Books

Learn the 'new' and


'unconventional
Core Areas New Areas
areas' from which
UPSC asks questions

Core Areas New Areas Exam Skills

We help you develop the ability to analyse the


question and find the correct answer using IETs
(Analytical Skills)

Download ClearIAS  App;


Enroll in ClearIAS Programs!
ClearIAS Prelims
Online Mock Test
Series

The Test Series You


Should NOT Miss!
Join now!

You might also like